wallnuts

none

wallnuts #Chat Talk

General Chat

Calc help needed from smart people Realized there's a lot of people on basil who are actually really good at math. Please help me on this problem [url=http://www.chegg.com/homework-help/questions-and-answers/-a-a-when-the-graph-of-y-x2-for-0-x-2-is-rotated-about-the-horizontal-line-y-l-the-volume--q1285015]Same as the one in this link[/url]. I got part b, which turned out to be (4/3). I'm doing this on webassign, which is quite picky with answers. Sorry if I seem lazy. I'm god awful at math and my professor, as smart as he is, cannot teach for crap. I'm only taking this class since it's a req. for nearly all science related majors.

General Chat

Smart people w/ calc knowledge needed The portion of the graph y = e^x between x = 0 and x = ln 3 is rotated around the y axis to form a container. The container is filled with water. Use n = 4 subintervals and midpoints to approximate the work required to pump all of the water out over the side of the container. Give your answer in decimal form. (Distance is measured in meters, the density of water is 1000 kg/m3, and use 9.8 m/s2 for g.) (Link to picture of graph if u need it): [url=http://tinypic.com/view.php?pic=2ntdzys&s=6]http://tinypic.com/view.php?pic=2ntdzys&s=6[/url] Now apparently the answer is roughly 19000J, but I need the exact decimal format. I tried it myself and i can't get 19000. Please help?

General Chat

Calculus help needed from smart people Never really ran into a problem like this. [url=http://tinypic.com/view.php?pic=1z48m5x&s=6]Blanking on this[/url] Now the question gives me that graph and tells me to "Fill in the missing coefficients in the integral". I'll try my best to make it as neat as possible: Integral: (0 to 4) (Pi) ((____)(f(x)^2) + (____)(f(x)g(x)) + (____)(g(x)^2)) dx hope that was neat enough. Not trying to evaluate it. Just fill in the blanks. Thanks.

General Chat

Calculus problem Smart people needed A particle that moves along a straight line has velocity v(t) = (t^2) (e^-t/3) meters per second after t seconds. How far will it travel during the first t seconds? Now according to my work and wolframalpha, the integral of that thing should be (-3e^-t/3)(t^2 +6t +18) According to my teacher its wrong. I have no friggin idea what's wrong. Please just take a look and see if it's right or wrong. I'd rather not argue with my professor with a wrong answer.